You are on page 1of 45

CRIMINAL LAW OUTLINE

I. INTRODUCTION
Preliminary Hearing - (1) probable cause to show crime was committed, and (2) probable cause to
show D did it.
Difference between civil and criminal sanction for same action - stigma. Goal of criminal law is to
punish, make restitution. Goal of tort law - compensate the victim.
Consent of injured maybe a defense in torts but often not in criminal aw. Similarities: proximate
cause, intervening cause, duty to act, self-defense.
"preponderance of evidence" vs. "beyond a reasonable doubt"
criminal law is slower to borrow from the civil statutes than the civil law is to borrow from
criminal statutes.
Desuetude - laws that are not enforced wither away.
II. CRIMINALIZATION
Factors affecting criminalization: 1) does it have moral dimension? 2) can the law be
enforced? 3) will the law promote respect for the law? 4) will the cost of enforcement be
greater than society's benefit? 5) will investigation methods necessary to enforce the law be
unreasonable or degrading? 6) does the crime have a victim?
A. CRIME AND MORALS
Sources of crime: 1. legislatures; 2. Constitution - treason; 3. administrative regulations;
4. martial law, military law, tribal law; 5. treaties (genocide); 6. common law crimes.
B. DEFINING CRIMINAL CONDUCT
1. Principle of Legality - violate specific rules made known in advance. Limit on the
government. If the Principle of Legality were strictly followed, statutes would be narrowly
construed, never retroactively. To some extent Pple of Legality included in Due Process.
2. Common Law Crimes
Something against the health, safety, and welfare of the public. (generic) Most jurisdictions
held common law crimes violate the Pple of Legality - unconstitutional, no advance
warning. Requires public harm, not private harm.
3. Legislature-made Crimes (a) Vagueness and Overbreadth
Vagueness - barred by Due Process of the 14th and 5th
Overbreadth - barred by 1st Amendment

Three bases for examining VAGUENESS: (1) does the statute in question give fair
and advance notice to those persons potentially subject to it - otherwise trap the innocent
by not providing fair warning? (2) does it adequately guard against arbitrary and
discriminatory enforcement - ad hoc basis for policemen and juries? (3) does it provide
sufficient breathing space for First Amendment rights? "men of common understanding"
must necessarily guess at its meaning and differ as to its application Connally Test
Standard used in applying vagueness statute - judicial gloss on statute. Is IL SC's
interpretation/refinement of the law to "imminent" diversion constitutional? U.S. SC does
not read vague statute as written, but as narrowed by the state courts -violates the principle
of legality.
Concerning the antinoise ordinance: "we do not have a general breach of peace" ordinance,
but a statute written specifically for the school context, where the prohibited disturbances
are easily measured by their impact on the normal activities of the school." Therefore not
vague. It is not overbroad as unduly interfering with First Amendment rights, including
right to picket on a public sidewalk near a school, since expressive activity is prohibited
only if it materially disrupts classwork. Grayned v. City of Rockford
Principle of Construction - each word is meant to mean something different. They are
mutually exclusive.
(b) Ex Post Facto and Bills of Attainder - Article I, '' 9 & 10. 9 - Congress, 10 states. Ex post facto - retrospective laws. Not permitted. Applied to conduct permitted
before the law. TWO ELEMENTS: 1) retrospective application 2) disadvantageous to
criminal accused.
Legislature cannot pass law today to make activities of yesterday a crime. Citizen responsible
for law at time of his actions. Punishment is rendered as it was at the time of
illegality. "Photograph of laws at time of crime."
Also cannot change retroactively procedures - only those procedures that protect a
substantial right - to disadvantage of the criminal accused. If legislature passes a new jury
law, that law would not apply to illegalities performed prior to the passage of the law.
Any law that causes a defendant to suffer a disadvantage, however, does not offend this
provision. The Supreme Court has distinguished between disadvantageous changes
affecting procedure and those altering substantial personal rights. The ex post
facto provision was not designed to limit the legislature's prerogative of altering both
remedies and procedures which do no involve matters of substance. Increasing the penalty
for a crime and imposing it retroactively violates the ex post facto guarantee. Altering the
method used to determine whether the death penalty was imposed was constitution. It did
not change the "quantum of punishment" attached to the crime. New Florida procedures
did not change the definition of the crime or the amount of proof necessary for guilt.
Parole laws do not violate ex post fact provision - no constitutional right to parole.
If act is not harmful to the criminal, then ex post facto does not apply.
Does ex post facto apply in judicial decisions? - only applies to legislative acts.

Bills of Attainder - legislative punishment of people who are readily identifiable. Cannot
single out group or individual for punishment. Bill of Attainder still is not identifiable. If
purpose of the bill is punishment, it is a bill of attainder. Not applicable to procedural
matters. (Originally, death sentence passed by legislature without benefit of trial legislative decree - Struck Down)
Example: Congress passed law that presidents' papers must be taken. Nixon claimed Bill of
Attainder - Nixon claimed impetus of the bill was punishment. Supreme Court did not
find for Nixon on issue of who owns presidential papers. Stated if Nixon had a financial
loss, compensation would be made.
Example: All lawyers must pay a $200 tax - bill of attainder. Neil Cohen must pay a $200
tax - bill of attainder. Reaches a particular person or easily identifiable group.
4. Substantive Due Process - limit on legislature's capacity to declare certain actions
criminal. Judicial second-guessing. Courts ask whether law serves a public purpose/interest. U.S. SC now says belongs to legislature. Does not deal with substantive due process
anymore. At the state level, still viable.
5. DOUBLE JEOPARDY CLAUSE - 5th Amendment bars prosecution for same crime twice,
includes merged crimes.
III. PUNISHMENT
A. RATIONALES
FOUR MOST COMMON RATIONALES:
1) retribution - societal revenge and expiation (atone for misdeeds through punishment). Still
commands the most public support. Claimed to suppress acts of private vengeance. May serve as
check against tyranny - you should not be punished unless you did something wrong.
2) incapacitation. Isolate the offender or disable him from offending. Requires you to predict
future behavior. When no longer a threat, then you can release.
3) rehabilitation - correction or reformation. Lose motive to do crime. Must predict future
behavior. Most in conflict with the other rationales.
4) special deterrence - exposing a criminal to sufficiently distasteful punishment, that individual will
lose the desire to commit crimes.
4) general deterrence - punishment will hopefully deter other would-be criminals from committing
crimes. Deterrence - important in cases like income tax evasion where system relies on selfassessment.
5)public education - inform the general public of what conduct is deemed socially unacceptable.
Regina v. Dudley and Stephens. An innocent person may not be killed in order to save the life
of another. Where the victim has not assaulted or otherwise endangered the killer, the killer has
not, by necessity, been placed in a position which permits him to kill the innocent victim. The
extreme necessity of hunger does not justify larceny, nor can it justify murder. While, generally, the

preservation of one's own life is a duty; in some cases, the highest duty may be to sacrifice
it. Neither can the temptation caused by hunger be called an excuse.
COMMENT: While this case actually discusses a defense to murder, necessity (which here
did not excuse the murder), the case appears here in the casebook more for its moral discussion of
why the defendants, unwillingly placed in a tragic situation, must be punished for their act. The
court notes that "Law and morality are not the same, and many things may be immoral which are
not necessarily illegal," but that law would be divorced from morality if the temptation to kill,
which arose, could be an excuse for the actual killing. Even if the temptation were a valid excuse,
who is to determine who must die so that the others might live? (The death sentence was later
commuted by the crown to six months' imprisonment.)
2. SENTENCING
Since 95% of those charged with crime enter a plea bargain, the only issue is sentencing.
TREND - jurisdictions adopt sentencing schemes which place the greatest emphasis upon the
nature of the crime which was committed and comparatively little upon the characteristics of the
particular offender - mitigates good behavior argument for parole.
TREND - Retribution became dominant over rehabilitation in the early 1970s.
Sentencing procedures have changed radically in the last few years. Judges decide sentences in all
50 states. Effect of statutes: REDUCE JUDGES' DISCRETION. Predominantly, legislature sets
both min and max sentence for a crime.
Right of allocution - right to speak at sentencing.
When a jurisdiction's jails and prisons are full, sentencing decisions must reflect this
reality. Emergency valve - early release due to unconstitutional overcrowding.
Sentences can be determinate and indeterminate. A sentence of 5-10 years is indeterminate.
Incapacitation, rehabilitation, and deterrence - require you to predict future behavior and
dangerousness.
"Equal Protection" has infrequently been successful in challenging sentences or sentencing
procedures. The key is finding an impermissible discrimination.
No federal parole. 15% reduction for good behavior.
PROBATION - suspend execution and suspend imposition. In the case of a probation violation,
the court may be bound by the original prison sentence it imposed in the suspended execution
model, but has no such limits in the suspended imposition model. Probationers enjoy a
conditional liberty that cannot be taken away without certain due process procedures.
FACTORS a FEDERAL COURT must consider in determining whether to impose a FINE: (1)
D's ability to pay, (2) burden that payment of the fine will impose on both the D and his/her
dependents, (3) whether restitution will be made by D to victim.

SCANDINAVIAN "DAY FINE" - fine a person the equivalent to a certain # of days pay for
work, e.g., 100 days. This system is fair to both rich and poor, since it punishes them
proportionately and equally in terms of relative loss.
FORFEITURE OF PROPERTY - another sentencing option.
RICO - bars a person from receiving income from racketeering or illegal debt collection
activities. 18 U.S.C. ' 1962. SC has upheld the forfeiture of drug proceeds used to pay for defense
counsel. Book proceeds restrictions against criminals violate First Amendment.
RESTITUTION - another alternative to sentencing.
DIVERSION - if does not violate law again, or meets community service requirement, charges will
not be filed or will be dismissed.
PROCEDURES - often the rules of evidence do not apply at sentencing hearings, making hearsay
evidence both admissible and frequently used.
Minnesota guidelines: 1) prescriptive - establish its own policies; 2) de-emphasize imprisonment
for property crimes, and emphasize for violent crimes; 3) in order to attack sentencing disparities,
established narrow sentencing ranges (e.g., 30 - 34 months, 50 - 58 months); 4) adopted "just
deserts" as the governing premise of its policies concerning who received prison sentences; 5) had
to take existing prison capacity as given, thus if lengthened one sentence area would have to
shorten in another. 6) forbade consideration of education, employment, marital status, etc. in
sentencing decision - to avoid discrimination against minorities, women, or low income groups.
SENTENCING MATRIX - requires the calculation of GRAVITY OF CRIME against
CRIMINAL HISTORY.
Victim's rights movement = harsh sentence.
B. EIGHTH AMENDMENT
The Eighth Amendment's cruel and unusual punishment clause is used to challenge the severity
of a sentence. The offender claims that a particular sentence is so harsh that it is
unconstitutional. These claims are rarely successful. It is not clear to what extent the eight
amendment embraces a requirement that sentences be proportionate to the offense.
The prohibition has three aspects: (1) limits the methods which may be used to inflict
punishment; (2) it limits the amount of punishment which may be prescribed for various
offenses; and
1. Status Crimes
(3) it bars any and all penal sanctions in certain situations - invalidate California law making it a
crime to be "addicted to narcotics." Robinson v. California. The punishment (confinement) is not
cruel, but the conduct should not be the subject of criminal sanction. Now laws incorporate
actions: "spreading" AIDS, "driving" drunk

Public drunkenness distinguished from status crime in Powell v. Texas. However, White said
some public drunks you cannot punish: 1) homeless and 2) involuntarily drunk.
2. Death Penalty
Gregg v. Georgia 1976 the U.S. Supreme Court reinstated the death penalty. (1) bifurcated
hearing, (2) list of aggravating circumstances, (3) tries to be predictable.
Fact that it does not deter does not matter; retribution is sufficient. Aggravating circumstances list
satisfies the concerns of Furman as to capricious and arbitrary administration. No clear way to
weigh aggravating and mitigating circumstances. Cannot have mandatory death penalty, must
allow for mitigating circumstances.
TREND: number of aggravating circumstances has been increasing.
MCCLESKEY V. KEMP, (1987). The death penalty is not unconstitutional because of statistics
demonstrating a tendency towards racial bias (victim) in its application. To prove an Equal
Protection violation, a person must prove he was the victim of purposeful discrimination, not a
discrim. factor.
Payne v. Tennessee (1991). May include victim impact statement. No standards for jury use.
It is unconstitutional for a state to mandate the death penalty for a certain degree or category of
murder; or for those murders unaccompanied by a few specified mitigating circumstances; or for a
sentencing judge to disregard as a matter of law relevant mitigating circumstances.
capital punishment must "be imposed fairly, and with reasonable consistency, or not at all," for "a
consistency produced by ignoring individual differences is a false consistency."
8th Amendment requires some proportionality - cannot have death penalty for nonheinous
felony. Minors can be executed.
3. Cruel & unusual punishment in other contexts
Courts have seldom held a punishment may be cruel due to excessiveness. Courts are seldom to
strike down sentence on proportionality grounds.
Harmelin v. Michigan 1991. No majority opinion today on PROPORTIONALITY. Harmelin
given a life sentence for drug possession. Today, if we have a grossly disproportionate sentence "way out of whack" - seven justices would strike. Scalia's approach would remove proportionality
for noncapital crimes.
Legislatures and judges can give pretty severe penalties.
C. BURDEN OF PROOF
1. MULLANEY v. WILBUR (1975). Maine homicide statute struck down that enabled state to
get conviction for murder without proving malice aforethought. D had to prove
manslaughter/heat of passion of automatically charged with murder. Court said this law denies
Due Process by shifting burden of proof onto D. For a constitutional law, the state must prove
each element of its case beyond a reasonable doubt. The prosecution must prove the absence of
the heat of passion on sudden provocation when the issue is properly presented in a homicide case.

2. In PATTERSON v. NEW YORK, (1977), the Supreme Court held that the modern "extreme
emotional disturbance" version of "heat of passion" could be interpreted as an affirmative defense
w/o a denial of DUE PROCESS. As an affirmat. defense, the burden of proof was on the D who
raised the defense. Separate defense from elements of the crime - consti. permissible.
3. Burden of production - responsible for producing evidence, government. D no
burden. Burden of persuasion - gov't must convince jury. Burden shifting - prima facie case
frequently. If D asserts insanity, must introduce evidence to support a prima facie case, then
burden shifts to gov't to get negative.
MARTIN v. OHIO. Common law rule placing burden for proving self-defense on D held
constitutional. Burden of production on D. An affirmative defense does not negate an
element. (Only Ohio and South Carolina have not abandoned the common law rule and
require the prosecution to prove the absence of self-defense when raised by the D. Could
juror feel reasonable doubt about self-defense but not preponderance of the evidence? Is
beyond a reasonable doubt for elements sufficient?
Indiana has been trying to write laws that say "possession of another's property is theft" then
allow defense for permission, emergency = changes lack of consent from element and
shifts burden to D.
4. Defenses fall under 1) exception, 2) defense, and 3) affirmative defense categories.
5. Mandatory presumptions (i) conclusive or (ii) rebuttable, and permissive inferences. When
conclusive, gov't does not have to prove intent only, e.g., unexplained presence. AfterFRANCIS v.
FRANKLIN (1985), prosecutors cannot use presumptions to prove an element of a crime; state
must prove each element beyond a reasonable doubt. Reasonable presumption allowed in
defense: a mandatory presumption that works to disprove an element (i.e. helps the D and does
not remove any of the state's responsibility to prove each element) is constitutionally
permissible. In other words, not all presumptions are invalid in criminal cases. On the
prosecution side, "presumption language" replaced with "inferences" today for jury
instructions. Permissive inferences do not violate the Due Process clause unless the conclusion is
one that reason and common sense would not justify in light of the proven facts.
Court does not decide issue of whether a mandatory presumption that shifts only a burden
of production to the D is unconstitutional.
PART TWO: CRIMES
I. THE CRIMINAL CODE
Exception - burden on D by preponderance of evidence. State does not have to prove
anything. Exception: law enforcement officer carrying a gun on school property. Burden of
persuasion on D.
Defense - burden on D to produce. Burden of persuasion - on gov't beyond a reasonable
doubt. To escape acquittal, gov't must negate defense beyond a reasonable doubt. Insanity, selfdefense.

Affirmative Defense - Burden on D for production and persuasion. 51% probability must be
established by D.
"Intoxication" is not a "defense" in TN, but it can get you off the hook like a defense
defense.

1 really is a

By default if you can't tell, it is a defense.


Rule of Lenity

2 rule of strict construction against the gov't. Close cases tilt in favor of D.

TREND = many states want an unbiased reading, interpretation of their criminal statutes just like
any other statute. No bias on vague provisions against the government.
II. BASIC ELEMENTS: the body and mind
Elements of a crime: 1) wrongful act - actus reus; 2) mens rea - evil mind; 3) resulting harm 4)
circumstances 5) causation (both factually and proximately)
Relationships: mind must actuate the act; the act must cause the harm.
Example: drive to someone's house intending to shoot/kill him, and runover by accident
instead. Link between mens rea and actus reus not present.
Split second in time fixes mens rea and actus reus.
The physical act and the mental state must exist at the same time.
A. ACTUS REUS - "voluntary" act as well as an omission where there was a duty to
act. "Voluntary act" has imbedded mens rea component.
Omission of duty - duty limits liability from the whole world. Duty to Act: (1) statute, (2)
contract, (3) relationship, (4) voluntary assumption of care/rescue attempted, (5) creation of peril.
kill someone while in convulsion - no crime. Acting like automaton or automatically - no actus
reus, not voluntary, must look inside criminal's head.
If you start to provide assistance, you must continue.
conscious exercise of the will. Contrast: (1) conduct that is not the product of the actor's
determination; (2) Reflexive or convulsive acts; (3) unconscious or asleep - unless D did so
purposely to engage in dangerous behavior.
Actus reus - required by 8th Amendment, Powell v. Texas leading the way. Yet TN has no actus
reus in our new criminal code. ???
B. MENS REA - the more mens rea elements, the more prosecution must prove.

Four mens rea elements in a modern penal code: 1) intentionally (in common law `purposefully,
willfully,' 2) knowingly - I thought it would happen, 3) recklessly - I thought it could happen and
went ahead, 4) criminal negligence - I should have known.
Higher degree of fault suffices.
strict liability crimes - have no mens rea element. There are few strict liability
crimes. (prosec. would like few or no mens rea elements, therefore strict liab. is
ultimate) Normal routine way - conviction based on circumstantial
evidence. Example: speeding (intent not important).
For knowingly, "reasonably certain" < "practically certain"
negligently - only mens rea element that does not have subjective component. Must
have gross deviation; otherwise only collect in tort law.
recklessly - aware of risk and disregarded it (subjective) also an objective element on gross
deviation from standard of conduct of a reasonable person.
2. Enumeration of Specific Intent Crimes
1) Solicitation: intent to have the person solicited commit the crime;
2) Attempt: intent to complete the crime;
3) Conspiracy: intent to have the crime completed;
4) First degree premeditated murder (where so defined by statute): premeditation;
5) Assault: intent to commit a battery;
6) Larceny and robbery: intent to permanently deprive the other of his interest in the property
taken;
7) Burglary: intent to commit a felony in the dwelling;
8) Forgery: intent to defraud;
9) False pretenses: intent to defraud;
10) Embezzlement: intent to defraud;
3. Malice - Common Law Murder and Arson
The intents required for "malice" crimes do not take the specific intent defenses. The common
law created this special mental state category especially to deny to murder the specific intent
defenses: voluntary intoxication and unreasonable mistake of fact.
4. General Intent - Awareness of Factors Constituting Crime

D must be aware that she is acting in the proscribed way and that any attendant circumstances
required by the crime are present. D need not be certain attendant circumstances exist; it is
sufficient tha she is aware of a high likelihood that they exist.
a. Inference of Intent from the Act
b. Transferred Intent - applies to homicide and battery. Does not apply
to attempt. Example: A shoots at B, intending to kill him. She hits C, only wounding
him. While A may be guilty of attempted murder of B, she is not guilty of attempted
murder of C.
5. Model Penal Code Analysis of Fault
a. Purposely, Knowingly, Recklessly
b. Negligently
Example: With liability for "knowingly makes a sale of an intoxicating beverage to a minor, the
Model Penal Code would require D know (1) sale took place, (2) beverage was intoxicating, and (3)
purchaser was a minor.
C. VICARIOUS LIABILITY & CORPORATE LIABILITY
Employer responsible for employee. Attribute the actus reus to another; requires NO PROOF
of ACTUS REUS by the D. Strict liability crime. He is responsible unless he is "powerless to stop
it." Differentiates from status crime. Duty is very high. Social policy to make him do everything
possible. Do not use reasonable person test. (?-why not) Areas for concern: public welfare,
public safety, food distribution/public health; environment.
UNITED STATES v. PARK
CEO of Acme found guilty of allowing rats to contaminate his food - actually just found in
warehouse. The only way a corporation, such as Acme, can act is through people.
The liability under the act is limited to corporate employees who have a "responsible share" in the
furtherance of the transaction which the statute outlaws. Moreover, the principle is recognized
that a corporate agent, through whose act, default, or omission the corporation becomes guilty of a
crime, is himself guilty of the crime. Finally, the Act imposes not only a positive duty to seek out
and remedy violations but also a duty to implement measures that will ensure that violations will
not occur. However, the Act does not require that which is objectively impossible. Ultimately, the
government must prove beyond a reasonable doubt the defendant's guilt.
The imposition of corporate liability does not affect the criminal responsibility of the actual
perpetrator. Hence, both the corporation and the guilty employee may be convicted of the
offense.
III. CRIMES AGAINST PERSONS
Contrast "common law" with modern provisions: 1) "malice aforethought" vs. NY v.
Patterson (extreme emotional distress); 2) fetus had to be born alive vs. crime of killing of a
fetus; 3) dead when heart stopped vs. brain death standard.

Common law murder: malice aforethought 4 types: intent to kill, intent to injure seriously,
commission of a felony, recklessness "depraved heart"
Common law manslaughter: no malice aforethought
A. HOMICIDE: TRADITIONAL
1. Malice Aforethought
a. Intent to Kill - yes - her goal was to kill Clarence.
b. Intent to Commit Serious Bodily Injury - yes (part of intent to kill)
c. Recklessness - yes, throwing a hand grenade in the park.
d. Felony Murder Rule - yes (killed Walters while trying to murder Clarence; also some
kind of weapons felony for exploding, possessing, etc. an active hand grenade; state statute
' 123.4, malicious destruction.
2. First Degree Murder
a. Willful, deliberate, premeditated
i. Willful - was intentional killing; purposeful; transferred intent.
Did it matter that she changed her mind and tried to retrieve the grenade before it
exploded? She no longer had the intent to kill at the time the victim was
killed. But was her change of mind too late?
ii. Deliberate - act performed with a cool mind
She appeared to plan the crime carefully; she concealed herself; no evidence of rage
iii. Premeditation - some advance reflection (however short) (See deliberation facts)
iv. Was he "lying in wait" when she hid behind a tree? if so, this may be evidence of willful,
deliberate, and premeditated action.
v. Use of poison also captures the three mens rea elements.
b. Felony-murder
Unless attempted first degree murder is a listed felony, is probably no felony in the list of
those qualifying for first degree murder.
3. Second Degree Murder
a.
b.
c.
d.
e.

Assuming there is malice aforethought - see above


Willful, deliberate, premeditated (at least one misssing/missing link)
Recklessness - reckless killing
Felony murder - not one of the five or so felonies listed; don't need intent to kill
Intent to injure homicide

4. Voluntary manslaughter
a. Heat of passion
i. Facts say Geogina hated Clarence
ii. Since reason for the hatred is unclear, don't know whether it would be legally sufficient
provocation. Worth a sentence to raise the issue. OBJECTIVE standard REASONABLE PERSON - do not consider particular sensitivities.
iii. Note: there was advance planning, so was there adequate cooling time? A brooding
case?

5. Involuntary manslaughter
a. Misdemeanor Manslaughter Rule
i. Misdemeanor weapons violation? Vandalism?
b. Criminal negligence - gross deviation from standard of care
i. Throwing hand grenade in Central Park
- others obviously around
- inherently dangerous act
ii. careless driving
B. HOMICIDE: MODERN STATUTES
1. Many varieties
2. Intentional homicide
a. Transferred Intent
3. Knowing homicide - 2nd degree murder in NY
a. Transferred knowledge?
4. Reckless homicide
a. Role of commission of felony, no mandatory or rebuttable prosec. presumptions
5. Voluntary Manslaughter
a. Now more SUBJECTIVE (was this person provoked?)
6. Criminally negligent homicide - replaces invol. manslaughter in NY
Difference between modern and common law statutes on homicide: 1) if you eliminate death penalty, may remove the
need for Murder 1 and Murder 2; 2) extreme emotional disturbance 3 heat of passion (change of language
only); 3) Common law was easier to prove recklessness - now unconstitutional for prosecution to
have mandatory or rebuttable presumptions; 4) eliminate felony murder rule; 5) no misdemeanor
manslaughter rule; 6) malice aforethought; 7) manslaughter very subjective, does not use as much
external standards for legally sufficient provocation. Does away with reasonable person.
Relation Murder 1 and Volun. Manslaughter: different interpretation of the same facts. Cooling
off period vs. brooding and building up.
Relation Murder 2 and Involuntary Manslaughter: degree of criminal negligence vs. recklessness fact in question.
Reforms of felony murder rule to accommodate criticism that accidents are put into the same category as more
blameworthy crimes: 1) felonies must be inherently dangerous; 2) proximate cause; 3) merger - we don't consider
battery as separate felony automatically triggering murder.
1.
2.

Murder 2 argument for Beth Kavel's killing of Harvey Cedars (p.B37) -- homicide during
felony murder, may qualify for recklessness using objective vs. subjective test
Manslaughter - heat of passion but lacks legally sufficient provocation.

Imperfect Self-Defense: leaves D open to volunatry manslaughter


TREND - aggravating circumstances group keeps growing.
2. OTHER CRIMES
A. ASSAULT

1. Traditional
a. Willful AND unlawful
b. Attempt to offer, with force or violence, to do a bodily hurt to another
c. (includes attempted battery and reasonable apprehension of immediate battery
assaults)
d. transferred intent applies.
2. Modern
a. attempts to commit a violent injury to another
b. commits act -> reasonable apprehension of immediately receiving a violent
injury.
c. some jurisdictions tack on ability to commit the assault.
Simple Assault - misdemeanor - intentionally, knowingly, recklessly (no negligently) requiring 1)
attempt to commit bodily injury, 2) apprehension of imminent bodily injury.
Felony Aggravated Assault - 1) causes serious bodily injury (includes pain and impairment of
mental faculty), 2) uses or displays a deadly weapon.
2 Categories of Assault: 1) carefully thought-out threat, 2) situational crime, bar room brawl
Example: case where fraternity brother rubs elbow of another. Argue against assault: not an
offensive touching.
B. BATTERY
1. Traditional
a. Willful AND unlawful
b. Use of force upon the person of another
2. Modern
a. Intentionally OR knowingly
b. Without legal justification
c. Or by any means - captures negligent battery
d. Causes bodily harm to an individual
e. makes physical contact of an insulting or provoking nature with an individual
3 kinds of BATTERY: (1) intentional, (2) criminally negligent battery, (3) aggravated battery (with
intent to rape, murder, serious bodily harm).
MERGER DOCTRINE bars conviction for both battery and murder.
C. RECKLESS ENDANGERMENT - something that creates an UNREASONABLE
RISK. NOT A SPECIFIC INTENT CRIME.
IV. CRIMES AGAINST PROPERTY
Thief - someone who disregards traditional property relations.

Custody - an owner places goods temporarily in someone else's control. Don't own it, don't have
the right to alter. Employer gives to employee to give to someone else.**
Possession - exclusive control, intent to exercise, separate from ownership. Larceny is crime from
someone's possession.
Bailment - Bailor entrusts goods to bailee. Usually by contract. Example: give clothes to
cleaners. A bailee has possession, no custody. Therefore, if cleaner keeps clothes, not guilty of
larceny. He already has possession.
Title - ownership = bundle of interests. Legally entitled, exclusive.
Abandoned property - belongs to nobody. Original owner relinquished interest.
1. LARCENY: (1) trespassory, (2) taking, and (3) carrying away, (4) of personal property, (5) of
another, (6) with intent to steal; and is classified as either grand or petit, depending on
whether value is > $50. Must take with evil mind from another's possession. Example: Third party to
servant (gets possession & keeps) then supposedly to master: no larceny.
Larceny by trick - obtain possession through deceit. Key: evil mind at time of acquiring
possession. "Constructive possession" fiction.
Larceny by bailee - where bailee acquires lawful possession then converts it. If bailee
misappropriates the entire bale without breaking it open, it was not larceny.
Steal files and return - no larceny, so long as files not depleted in value.
Asportation - "moving every part of it" "carrying away". Most modern codes do not use
this concept.
Owner can commit larceny of his own property: rental car repossess.
Prove intent with circumstances. JOYRIDING - temporary taking.
MODERN STATUTES (1) differentiate misdemeanor petit larceny from felony grand
larceny, (2) cover both theft of property and services, (3).
Nature of property: must be tangible personal property. A deed, representing real estate, or
a contract, representing an intangible right to performance, could not be stolen. Also in
common law one could not steal intangibles such as stocks, bonds, checks, promissory
notes, all of which are documents.
VALUE - often divided into grand larceny and petit larceny; inflation tilts more toward
grand larceny.
2. Embezzlement - one, who through position of trust, obtains lawful possession of another's
property, then turns evil and converts. (agents, servants, employees)
Servant who takes property intended for his master from a third party, the servant acquires
possession, and his misappropriation is not larceny, it is embezzlement. If he puts money

in the cash register first, then takes it, it is larceny providing the cash is in the register long
enough for master to acquire possession.
a. Fraudulent Breach of Trust - fiduciaries;
b. Fraudulent conversion - anyone, but must have clean mind.
At common law, embezzler punished less than larcenist due to expectation for less violent
behavior. Modern statutes treat the same.
3. False pretenses - obtain ownership (title) by deceit. (Cousin of larceny by
trick) Misrepresentation of past or present fact such that (1) one obtains title, title passes by
trick,affirmative misrepresentation; (2) fact must be material. Mens rea - must know that it is false
and must (3) intend to defraud. Example: forgery.
4. Extortion - obtain property by threats to do something in the future, ordinarily not involving
physical harm; does not have to be threat of violence. "blackmail" Common law: public official
taking a bribe.
How Obtained

What Person Obtained


Possession
Title
Stealth
larceny
Deceit
larceny by trick
False pretenses
Force/Threat
extortion, robbery
XXXXX
Rightful, then convert larceny by bailee
American way
5. Robbery - (1) trespassory (2) taking and (3) carrying away of the (4) personal property OF
ANY VALUE (5) of another (6) with intent to steal it PERMANENTLY + (7) taken from the
person or his presence of the other and (8) accomplished by force of putting in fear. "mugging"
animus furandi - intent to steal; does not need intent to convert to one's own use, only deprive the
owner of the property.
Value of property not affected
Negated by: (1) honest mistake to claim of ownership; (2) takes with intent and ability to
return in a reasonable time; (3) practical joke - still not blameless.
Person: includes homicide victim,
Violence line between larceny and robbery: not easy to draw. Include "snatching"
cases. May also render victim helpless by blow or drugs.
Aggravated/Armed Robbery: Use of a dangerous or deadly weapon, inflicted serious bodily
injury, had an accomplice, or used dangerous instrumentality.
CONTRAST with Larceny from person - "purse snatching;" elderly woman falls and breaks
hip, but not enough violence to constitute robbery. Pickpocketing can be larceny or
larceny of the person.

Classification DEPENDS on POTENTIAL HARM not acutal harm to the victim.


6. Receiving Stolen Property "fence"
a. receives or conceals
b. stolen property
Mens rea: (1) knowledge that stolen, and (2) intent to deprive owner PERMANENTLY
requires more than "put in my greasy hands"
If thought goods were legit, then find out stolen and keep, then "concealing" catches you.
Ambiguous what "stolen" means - false pretenses, extortion, etc.
"Knowledge" does not mean certainty, only belief.
Not guilty - if you think it is your property, or purchase with intent to return to owner.
ordinarily - treated the same as larceny.
7. Consolidated Theft Crime - embezzlement, false pretenses, fraud conversion, receiving stolen
property, extortion ... now all rolled up (in Tennessee) in theft crime 39-14-103. "knowingly
obtains or exercises control over property without the owner's effective consent." Old distinctions
- custody, possession - eliminated. False pretenses - handled by "effective consent."
8. BURGLARY - common law (1) breaking and entering of the (2) dwelling place of another (3)
in the nighttime (4) with the intent to commit a felony.
entry through pre-existing open door or window: no breaking. Opening of a door closed
but not locked: sufficient breaking. Entry by fraud or force: constructive breaking.
MODERN CODES: no "breaking" - use "unlawful" instead; kept "entry"; replace
"dwelling" with any building, no "night". Permit affirmative defense that the building was
abandoned. Aggravated burglary - burglary of a habitation.
Access crime not subject to MERGER. Can be convicted of both burglary and rape.
9. INCHOATE CRIMES - "foreplay" of criminal law: 1) assault, 2) burglary (real purpose is
theft or kill), 3) attempt, 4) solicitation, 5) conspiracy.
Focus on dangerous person - actor, authorizes law enforcement to intervene.
A. ATTEMPT
1. TRADITIONAL
a. intent for criminal conduct - prevents us from having "attempted negligent or reckless
crimes." If intent present, then not negligent or reckless.
b. specific intent for the underlying crime
c. an overt ACT beyond the PREPARATION stage

d. No ABANDONMENT defense. Once you cross the perpetration line (requisite


acts with criminal intent), you are guilty, regardless of whether you change your
mind.
2. MODERN STATUTES
a. Permit renunciation defense (religious conversion): VOLUNTARY and
COMPLETE ABANDONMENT.
i. Change mind when you see cops are present - no renunciation or negation of your intent.
ii. Involuntary abandonment is no defense.
iii. Burden of proof on D to show a manifested act to prove his renunciation.
iv. Decision to postponse crime until another time 4 abandonment.
3. OVERT ACT
The overt act must be beyond mere preparation for the offense. Look at all circumstances for
corroborative evidence.
a. Proximity Test - have done everything he believes necessary to bring about
result; physical and time proximity to intended crime. (Used most often in
TRADITIONAL jurisdictions) (the more remote in time and space, the less likely a court
will find attempt)
b. ordinary course of events would lead to crime except for the intervention of some
extraneous factor; likelihood that crime will be committed. (the more likely D's act will
result in a crime, the more likely a court will find attempt).
c. Equivocality Test/Res Ipsa Loquitur Test: If D's acts do not on their face indicate
criminal intent, some courts will not convict for criminal attempt. Act itself is evidence
of unequivocal criminal intent - *** different from other types of crime **;
d. manifested "dangerousness" of the person; (the more clearly the D's conduct has been
manifested, the more likely a court will find attempt)
e. the seriousness of the crime. The more serious the crime, the further back in time the
court may reach to find an attempt.
f. MODEL PENAL CODE Approach - "SUBSTANTIAL STEP": possession of
materials to be employed in the crime, unlawful entry, lying in wait, etc. The "step" must
be "strongly corroborative of the actor's criminal purpose. ALSO uses act to establish
INTENT.
4. Defenses to Liability for Attempt
LEGAL IMPOSSIBILITY - where D has done all the acts intended, but his conduct does
not constitute a crime. Example: person who erroneously believes he is in a country
forbidding alcohol sales, buys or attempts to buy some liquor. Legislature must define as
crime first. Self-defining criminality (e.g., picking nose)
FACTUAL IMPOSSIBILITY - D is unable to accomplish his criminal purpose because of
facts unknown to him. Not a legal defense. Example: pickpocket who finds anempty
pocket. Still charged with "Attempt"
MIXED FACT-LAW IMPOSSIBILITY - most troubling to the courts, mixed results,
generally not a defense. D did all acts intended in an area covered by criminal statutes,
but cannot be convicted of the substantive offense because of a factual mistake pertaining
to the legal character of a collateral matter. Example: A bribes B on the erroneous belief
that B is a juror.

Example: Police sell "goods" to C, who mistakenly believes they are stolen. Can be charged
with attempt to receive stolen goods.
People v. Jaffe D purchased certain goods in the mistaken belief they were stolen. Ct of
appeals reversed conviction of attempt because "if the accused had completed the act
which he attempted to do, he would not be guilty of a criminal offense." Jaffe analysis
rejected in cases where person thought the cigarette he smoked contained marijauna, the
drugs he sold or received contained heroin, the girl with whom he had sex was alive, or the
man he shot was alive. One form of LEGAL IMPOSSIBILITY DEFENSE.
Modern Penal Code rejects Jaffe because it does not look at actor's "intent" or
"purpose" but rather at situation wholly at variance with the actor's
beliefs. Attempt liability should be imposed. Actor has manifested his
dangerousness and clearly demonstrated his criminal purpose.
Wilson D changed a draft from 2.50 to 12.50. He did not change wording "two dollars and
50/100 dollars" or the marking "ten dollars or less." But forgery statute requires a change
in a material part of the draft. D may have thought he was committing a crime, but if he
did it was not because he intended to do something that the criminal law prohibited but
rather because he was ignorant of the material alteration requirement of the crime of
forgery. Not guilty of either forgery or attempt to commit forgery. Application of
Principle of Legality: D did not intend to do anything that had been made criminal, and
what is not criminal may not be turned into a crime after the fact by characterizing his
acts as an attempt.
INHERENT IMPOSSIBILITY - D employs means a reasonable man would view as totally
inappropriate to the objective sought. Example: voodoo doctor put pins in a doll to
commit murder. (Others argue he might try a gun next time).
5. Double Jeopardy prohibits prosecution for both crime and attempt of that crime
6. Serves the important function of allowing law enforcement officers to INTERVENE in time to
prevent crime.
7. In Tennessee, attempt - punishment 1 level below crime attempted. An attempt indicates
disposition toward such activity - REHABILITATE. General deterrence - minor function of the
law of attempt.
voluntas reputabitur pro facto - the intention to be taken for the deed.
insidiator viarum - one who lies in wait to commit an offense
B. SOLICITATION
MERGER DOCTRINE - bars conviction for both solicitation and conspiracy in many
jurisdictions.
Modern statutes allow a renunciation defense. Minor crime. Does not have to be
successful. NOTHING ELSE NEED BE DONE*** Ordinarily occurs before you cross line for
attempt. Must have intent for committing the crime.

Every time solicitation is successful = conspiracy, as long as both are serious (not an undercover
cop). Solicitation = attempted conspiracy.
In TN, punished two levels below the crime.
C. CONSPIRACY - punishes group interest; NO MERGER bar against prosecuting for
substantive crimes. In Tenn, punished one notch below crime intended, which can increase
sentence by 75%.
1. TRADITIONAL
a. Intent to enter into an agreement; may be inferred from conduct; intent cannot be
inferred from mere knowledge - person who sells someone a gun.
b. Intent to accomplish some criminal or unlawful purpose or accomplish some lawful
purpose by unlawful means. CORRUPT MOTIVE DOCTRINE - usually applied to
offenses malum prohibitum. Parties must know their objective was criminal.
c. agreement/UNDERSTANDING between 2 or more people; agreement itself is the
actus reus.
d. covers any act injurious to public health, morals, or obstruct justice
e. conspiracy for general crime, ordinarily not a serious offense. conspiracy for specific
crime, e.g., bribing a public official, punished at one level below underlying crime.
f. PINKERTON RULE - charge other members with remote offenses caused by one
conspiracy member if (1) further objectives of conspiracy and (2) foreseeable consequence.
g. Must be meeting of TWO or MORE MINDS. Eliminate conspiracy convictions where
one party was an undercover cop.
2. MODERN
a. object of conspiracy be a specific proscribed offense
b. Intent to achieve the objective of the agreement; must be established for each individual
defendant
c. overt act in furtherance of the conspiracy, but mere preparation will usually
suffice. (Contrast with "substantial step" of attempt)
d. Reject Pinkerton Rule; use accomplice liability approach
e. MPC Unilateral conviction approach.
f. A few jurisdictions permit renunciation as a defense to the conspiracy charge itself.
(1) VENUE RULE - prosecutors are allowed to bring charges in any location (venue)
where an act in furtherance of the conspiracy occurred. Thus, Dr. Spock charged in Seatlle
with conspiracy to burn draft card for speech given in Boston.
(2) CIRCUMSTANTIAL EVIDENCE - marginally relevant, circumstantial evidence is
often admissible.
(3) HEARSAY ALLOWS - relaxed evidentiary rules; any statement of a conspirator in
furtherance of the conspiracy is admissible against any co-conspirator.
(4) Understanding Element - oral agreement with faded edges. Does it include an
escape? All subjective - may not agree on all parts. It takes two to conspire. We look at
theintent in both their heads. If one thinks in conspiracy and other doesn't = no
conspiracy. Does each know they are in a conspiracy with unknown others? In

prosecution, we look at what they intended on an issue they never thought


about. Agreement - who is in it, when does it begin and end, rarely written.
POSSIBLE TO CRIMINALIZE THOUGHTS - Axle Sally may not have done
anything in the CB radio conspiracy.
(5) FORMS of CONSPIRACY: (1) Spoke Wheel (hub and spoke). Pinkerton Rule
especially important for this form. (2) Chain Link - we can draw conspiracy circles around
part or all of the links.
(6) PINKERTON RULE: each member of the conspiracy is responsible for crimes
committed by every other member of the conspiracy that are foreseeable. To determine
number of conspiracies in a group - jury question based on "understanding" - immense
consequences. Example: charged with conspiracy to sell drugs and murder. (Is it
foreseeable?) Problem on p.B87: Axle Salley could be charged with conspiracy to obstruct
justice and 2nd-degree/reckless murder (of the hitchhiker). The substantive crime must
be reasonably foreseeable to be charged with it. Great expands liability, but unfair
element.
MODERN APPROACH: TREND: Most jurisdictions have rejected
the Pinkerton rule. In these, liability for the substantive crime must be proved by
the regular rules of accomplice liability rather than through the actor's participation
in the conspiracy. In order to be guilty of the substantive offense, the accomplice
must, with few exceptions, have aided, encouraged, counseled, abetted, assisted, or
in some other manner furthered the commission of the crime.
(7) Where two persons are charged with conspiracy, an acquittal of one is generally held to
bar the conviction of the other. The acquittal of all people with whom D is alleged to
have conspired precludes conviction of the remaining D.
(8) Generally solicitation proceeds conspiracy, but not always: e.g., CB radio conspiracy has
no solicitation.
(9) Comparing CONSPIRACY and ATTEMPT: (1) Conspiracy may require an act;
attempt always requires an act. (2) For conspiracy, an insignificant act may qualify as an
overt act; reason: proves intent, more than just talking about it. For attempt, the overt act
must cross the line to the perpetration stage. (3) Conspiracy penalizes group activity, but
the model penal code has a unilateral approach when dealing with undercover agents.
(10) Substantive crime must be related to conspiracy to be charged with it.
DEFENSES TO CONSPIRACY:
1. ABANDONMENT OR WITHDRAWAL - (i) a defense only for crimes committed
after the withdrawal. May still be charged with conspiracy and with substantive
crimes committed prior to withdrawal. (ii) starts the running of the statute of
limitations and terminating the co-conspirator's exception to the hearsay rule for
statements made subsequent to withdrawal. (iii) must effectively communicate to all
conspirators and in time for them to abandon their criminal purpose. Communication
satisfy reasonable man test. Some statutes permit notification to police in lieu of other

conspirators. (iv) few modern statutes permit renunciation to be a defense to charge of


conspiracy itself.
2. WHARTON'S RULE - a conspiracy conviction will not lie where concerted action is
necessary to commit the substantive crime which is the objective of the conspiracy and no
more than the minimum number of persons required for the substantive offense has
participated in it. Rule of statutory construction. Can't commit adultery by yourself or
gamble - requires 2 or more - therefore, if only two people, charge with gambling but not
conspiracy to gamble.
3. Factual impossibility is no defense.
4. Violates First Amendment right of free speech. Statute may suffer from vagueness and
overbreadth.
5. In Tennessee cannot have conspiracy for class C misdemeanor, because conspiracy
penalty is one level below substantive crime. Can have conspiracies for class A & B
misdemeanors as well as for felonies.
VII. PARTIES TO CRIME
A. PRINCIPALS AND THEIR HELPERS - Accomplice liability permits you to determine
who is guilty of the substantive crime without regard to the existence of a conspiracy.
Two ways to analyze culpability of other parties: (1) accomplice liability, and (2) conspiracy
with Pinkerton Rule. Without Pinkerton Rule, conspiracy law is of limited use. It would
not tell you who is guilty of the substantive crime.
1. THREE CLASSIFICATIONS OF THOSE INVOLVED IN A CRIME: (1) those who incite
or aid another in committing a crime; (2) actual perpetrators of the crime; (3) persons who aid the
perpetrator after the crime has been committed. Persons giving post-crime aid are often punished
less severely than those furnishing pre-crime aid. In common law the distinction between the three
groups was sharply drawn; now tendency to assimilate inciters and aiders (often lumped together
under the heading accomplice).
2. COMMON LAW/Traditional CLASSIFICATION FOR PARTIES TO A
FELONY: (1) Principal in the first degree - one who personally or with non-human
instruments or through an innocent agent actually committed a crime. For any felony, there could be
more than one principal in the first degree. (2) Principal in the second degree - originally called
"accessory at the fact" and also "aider" and "abettor" - one who aided, assisted, counseled,
commanded, or encouraged the principal in the first degree or stood ready to render aid if
needed. (3) Accessory before the fact - one who ordered, counseled, encouraged, or otherwise
aided the commission of a felony, but who was not present at the crime. (4) Accessory after the
fact - one not involved in the commission of the felony who concealed the felon or otherwise
aided the felon to escape detection, arrest, trial, conviction, or punishment. (1) - (3) are subject to the
same punishment; (4) is subject to less severe sanction. Why? - no role in commission of the substantive
crime; only obstructing justice.

3. COMMON LAW CLASSIF. FOR PARTIES TO A MISDEMEANOR: all parties involved in


a misdemeanor were considered principals, except that an accessory after the fact to a
misdemeanor was not guilty of a crime.
4. TRADITIONAL ACCOMPLICE liability for crime of another: (1) intentionally aid, encourage,
advise, hire, counsel, or conspire with or otherwise procure the other to commit the crime; (2)
person liable under (1) also liable for any other crime if reasonably foreseeable by him as
a probable consequence of committing or attempting to commit the crime intended
(PINKERTON); (3) person may be charged with the crime although the person who directly
commits the crime is acquitted or convicted of a different crime, or is otherwise not subject to
criminal prosecution.
5. SOME MODERN ACCOMPLICE STATUTES have rejected the reasonably foreseeable probable consequence (or "natural and probable consequence") approach and limit liability to
more direct participation in the crime: (1) solicits or commands such other person to commit the
offense; (2) aids, counsels, or attempts to aid such person in planning or committing the offense;
or (3) has a legal duty to prevent the commission of the crime, and fails to make proper effort to
do so. Responsible for conduct of another -solicitation, innocent agent, or omission.
6. MODERN STATUTES - (1) use words "party, principal, or accomplice" rather than "principal
of first degree," etc. (2) mens rea element is intent to commit or aid the commission of the
crime. Therefore, just hiding someone is not enough.
Example 1: Plumber's beating: 1. first-degree principal under common law; 2. "offense by
own conduct" - modern provision. Example 2: Roomie is 2nd degree principal under
traditional; "acting with intent to aid" - modern. Example 3: If junior unaware that Roomie
is outside to help, Roomie not guilty because he is not responsible for junior's action. He
did not provide any solace/aid. If Roomie actually does some act, e.g., waits on a customer
with intent to prevent discovery, then he is guilty.
7. LIABILITY OF SELLER FOR THE CRIMINAL ACTS OF THE BUYER: (1) One view,
used especially for more serious offenses, is that the seller is liable if he knowinglyfurnishes
assistance which he knows will facillitate the commission of the crime.
(2) MODERN TREND - seller is liable only if he knowingly supplies items that he knows will be
used for a crime and also intends (or has "purpose") to promote or facilitate the crime. (3) a few
jurisdictions adhering to the "purpose" standard have created the crime "criminal facilitation" carrying a less severe sanction than accomplice. Some have enacted a "compounding an offense"
statute.
8. DEFENSE of WITHDRAWAL: (1) former accomplice must repudiate his prior aid and/or
communicate his withdrawal to his fellow criminals in time for them to abandon the planned
crime. Furthermore, some jurisdictions will not accept the withdrawal defense if it was motivated
by the increased likelihood of detection. Some jurisdictions require the withdrawer take active
steps to frustrate the crime's accomplishment.
VIII. DEFENSES TO CRIME - TREND in Tenn. & 60% other jurisdic. - common law
defenses eliminated; defenses are statutorily prescribed. Public policy - give person a break - they
are less morally culpable.

FOUR CLASSES OF DEFENSES: (1) Protection of (societal recognized) Competing Interests self-defense, defense of others, lawful arrest, crime prevention, defense of property or
habitation. Person must reasonably and honestly believed force was necessary; many jurisdictions
require belief of imminent attack; not available to original agressor unless his nondeadly force
was met with deadly force or he withdrew from the encounter.
(a) Some jurisdictions using "alter ego" approach say rescuer's right to use force is only as
great as the attacked person's right to defend himself. Others say only an "honest" belief is
required; others require "honest (SUBJECTIVE) and reasonable (OBJECTIVE) belief."
(b) use of nondeadly force permitted for defense of property other than a dwelling house
from imminent, unprivileged intrusion or theft. Also used to recapture property
wrongfully taken. Some modern codes extend defense to property of others. Some
jurisdictions require honest beliefs and others hold they must also be reasonable.
(c) deadly force permitted for defense of dwelling. Deadly force is generally limited to
preventing dangerous felonies.
(2) Necessity or Duress/Choice of Evils - violate the criminal law in order to avoid a greater
harm. Difference is whether a natural physical force (necessity) or force from a third party (duress)
is the source. Courts generally hold that a D must select the least harmful alternative. Common
law required that the harm faced be imminent.
(3) Accused disproves or negates the mens rea element: infancy, intoxication, mistake of fact or
law, and consent. No crime. In certain situations not permitted to raise these defenses on public
policy grounds.
(4) Ignorance of the law - void for vagueness, or ignorance of certain legal principles.
Insanity - treated later.
COHEN's PRESENTATION: 1. Partial Defense - drops offense to lower level, not complete
excusing. E.g., voluntary manslaughter, imperfect self-defense - use excessive force, preemptive
strike. 2. Negative Element (negate mens rea) 3. Complete Excuse. Example: insanity, selfdefense.
Doctrine of relative innocence - everyone at fault, but someone is less at fault than
another. (Applications: Self defense, human life has more value than property, to facilitate
arrest of criminals).
Burden of proof: sometimes on D, sometimes shifts to prosecution, MOST
JURISDICTIONS/TREND put some burden on D.
A. TEN DEFENSES
1.

Self-Defense - like tort concept, complete defense. Objective - amount of force used must
be reasonable + Subjective - I must honestly believe force is necessary. Responding to
unlawful force (maybe even by police).
Do not need to consider running away as an option.

Escalation - person with white hat changes instantly to excessive force, and loses right to use
self-defense. forget history, take photograph of the time when force is applied. If using
reasonable force to respond to excessive force, then you have self-defense
defense. Sudden escalation - aggressor then has right to self-defense.
Minority of jurisdictions have Safe Retreat Rule - before using deadly force requires safe
retreat if possible unless in your home or sometimes in your place of business. Majority
rule - no retreat is necessary.
Retreat vs. withdrawal - person who presses on becomes the aggressor. Other party must
stop.
2. Defense of Others
a. Does not require special relationship.
b. "Traditional" approach - "Stand in the Shoes Rule" Privileged to use the same force for
defending a 3rd party as for self-defense. MAJORITY jurisdictions - mistake is no
excuse. Minor trend - allow for mistake. Model Penal Code - allow for "honest"
mistake. This one encourages people to help. In Tenn, reasonable rescuer rather than the
original victim's perspective.
3. Defense of a Dwelling
a. Nondeadly Force is justified whenever reasonable to prevent entry or attack upon one's
dwelling.
b. Deadly Force is permitted in some jurisdictions in defense of habitation where it
appears reasonably necessary to save the habitation from destruction, or believe felony will
be committed, or tumultuous entry + personal danger. Otherwise, reasonable force is
permitted.
c) Common law and some jurisdictions require the person claiming the defense to have first
requested the trespasser to leave.
d) Minority of jurisdiction have extended the defense of habitation to include places of
business.
4. Defense of Property - law permits reasonable use of non-deadly force to protect their
personal property or property other than a dwelling house from imminent, unprivileged intrusion
or theft. Such force is also permissible for immediate recapture of property reasonably believed to
be wronfully taken. Some modern codes extend these privileges to the protection of another's
property. Deadly force may not be used.
Some jurisdictions require honest beliefs, others require reasonable as well.
5. Defense of Crime Prevention

a. Nondeadly force - A person who honestly, and in most jurisdictions, reasonably


believes that a felony or breach-of-peace misdemeanor is being committed or is about
to be committed in his presence may use reasonable force to prevent its commission.
b. Deadly force is generally limited to preventing dangerous felonies.
6. Use of Force to Effectuate Arrest
a. By Police Officer
The use of deadly force to apprehend a fleeing felon constitutes a seizure. the force used to
effect a seizure must be reasonable. Deadly force is reasonable only when the felon
threatens death or serious bodily harm and deadly force is necessary to prevent his
escape. [Tennessee v. Garner (US 1985)]. Thus, a police officer cannot use deadly force to
apprehend an unarmed, nondangerous felon; but an officer may use deadly force to
prevent a felon from escaping if the felon poses a threat of serious bodily harm to the
officer or others. 4th Amendment violation.
An officer may arrest someone without a warrant for a crime that he observes.
OBSERVED CRIMES: Misdemeanor - no authority; Felony - probable cause D did it and
probable cause crime was committed.
b. By Private Person
A private person has the same right to use force to make an arrest as a police officer or one
acting at the direction of a police officer, except that the private person has a defense to the
use of deadly force only if the person harmed was actually guilty of the offense (i.e.,
felony) for which the arrest was made. It is not enough that it reasonably appeared that the
person was guilty. A private person has a privilege to use nondeadly force to make an
arrest if a crime was in fact committed and the private person hasreasonable grounds to
believe the person arrested has in fact committed the crime.
A citizen may arrest someone without a warrant for a crime that he observes.
OBSERVED CRIMES: Misdemeanor - no authority; Felony - probable cause D did it and
a felony was actually committed. No leniency for mistake about felony.
7. Resisting Arrest
a. Right to Resist Person Not Known to Be Police Officer
An individual may lawfully repel, with deadly force if necessary, an attack made by a police
officer trying to arrest her if the individual does not know that the person is a police
officer.
b. Right to Resist Known Police Officer
OLD RULE: May a person resist arrest if the person attempting to make the arrest is
known to be a police officer? Yes, nondeadly force may be used to resist an improper
arrest even if a known officer is making that arrest.
NEW RULE: must submit to arrest whether lawful or not.
c. When using excessive force whether by officer or citizen

8. Necessity
Conduct otherwise criminal is justifiable if, as a result of pressure from natural forces, the D
reasonably believed that the conduct was necessary to avoid harm to society exceeding the
harm caused by the conduct. The test is objective; a good faith beilef in the necessity of
one's conduct is insufficient. Causing the death of another person to protect property is
never justified. The defense of necessity is not available if the D is at fault in creating the
situation requiring that she choose between two evils.
Example: Jettisoning of cargo at sea during a violent storm, if necessary to save lives,
would not constitute criminal damage to property. On the other hand,
jettisoning some members of the crew to save the cargo would never be
justifiable.
a. Duress Distinguished
While duress involves a human threat, necessity involves pressure from physical or natural
forces.
Example: A points a gun a B and threatens to kill B if she does not break into C's house
and steal food. B does as she is told. B may raise the defense of
duress. If, however, B is a starving victim of a plane crash in a desolate
area and commits the same act, she has the defense of necessity.
9. Public Policy
A police officer (or one assisting her) is justified in using reasonable force against another, or
taking property, provided the officer acts pursuant to a law, court order, or process
requiring or authorizing her to so act. Example: executioner.
10. Domestic Authority
The parents of a minor child, or any person "in loco parentis" with respect to the child, may
lawfully use reasonable force upon the child for purpose of promoting the child's welfare.
B. THE DEFENSE OF DURESS
A person is not guilty of an offense, other than homicide (in some jurisdictions), if he
performs an otherwise criminal act under the threat of imminent infliction of death or
great bodily harm on himself or a third party. Act committed under duress is
termed excusable rather than justified: society condones the acts but does not encourage
them.
Example: If Art forced Betty to run down a police officer at a police roadblock, killing the
police officer, she would be guilty of a criminal homicide.
C. OTHER DEFENSES - most often mens rea knowledge element, but also intent
1. Mistake of Ignorance of Fact
a. Mistake Must Negate State of Mind
Ignorance or mistake as to a matter of fact will affect criminal guilty only if it shows that the
D did not have the state of mind required for the crime.

Defense must be raised by D. Let's you be guilty of what you thought you were doing. For
example, can be convicted of marijauna possession when in fact you possessed cocaine.
b. Requirement that Mistake Be Reasonable
1) Malice and General Intent Crimes - Reasonableness Required
2) Specific Intent Crimes - Reasonableness Not Required
Example: take an umbrella by accident - no larceny.
c. Strict Liability Crimes - Mistake No Defense
Since strict liability crimes require no state of mind, mistake or ignorance of fact is no
defense to them.
2. Mistake or Ignorance of Law
a. General Rule - No Defense
b. Mistake or Ignorance of Law May Negate Intent
If the crime is one that requires knowledge of some aspect of the law, ignorance or mistake
as that aspect of law will negate the state of mind. Such crimes are uncommon. Those that
exist generally require awareness of some aspect of law other than the existence of the
statute making the act criminal.
Example: Statute prohibiting the sale of a pistol to a convicted felon. If A believed the
buyer to be only a convicted misdemeanant (because she thought assault was a
misdemeanor), she lacked the state of mind required for the crime.
2) B, who has had her car repossed by a loan company, honestly believes she is still the
lawful owner of the vehicle and is lawfully entitled to possession of it. She sees it sitting in
a parking space in front of the loan company office and takes it. Even if B is wrong about
her right to take the automobile, she is not guilty of larceny because she lacks the requisite
specific intent.
c. Exceptions
- Statute Not Reasonably Available (Unpublished)
- Reasonable Reliance Upon Statute or Judicial Decision: any written opinion.
- Reasonable Reliance Upon Official Interpretation or Advice of Admin
Agency
- COMPARE: Reliance Upon Advice of Attorney is NO DEFENSE
3. Consent - may negate element of the offense. (Rape, kidnapping, theft); assault and
battery - boxing and wrestling (implicit).
a. Requirements of Consent as a Defense
(1) authoritative;

(2)
(3)
(4)
(5)

consent was voluntary and freely given;


party was legally capable of consenting;
No fraud was involved in obtaining the content;
Cannot exceed the scope of consent.

4. Forgiveness by the Injured Party No Defense


5. Criminality of the Victim is NO DEFENSE
6. Entrapment
Majority SUBJECTIVE approach: did gov't induce someone not predisposed to commit
crime?
Requires proof of two elements: (i) the criminal design / criminal intent originated with
law enforcement officers; (ii) the D must not have been in any way predisposedto
commit the crime.
a. Offering Opportunity to Commit Crime is NOT ENTRAPMENT
It is not entrapment if the police officer merely provides the opportunity for the commission
of a crime by one otherwise ready and willing to commit it.
Example A, an undercover police agent, poses as a narcotics addict in need of a fix. B
sells narcotics to A. Does B have the defense of entrapment? No. By
posing as an addict, A merely provided an opportunity for B to commit the
criminal sale.
b. Inapplicable to Private Inducements
A person cannot be entrapped by a private citizen. Inducement constitutes entrapment only
if performed by an officer of the government or one working for him or under his control
or direction.
c. Inconsistent with Other Defenses
If a D denies her participation in the offense, she has elected not to pursue entrapment and
is not entitled to raise the issue, even if the facts would otherwise permit her to do so.
d. Practical Difficulties of Entrapment: (1) Putting Predisposition at Issue Extended inducement raises question. Was it instilled by officers? A mere "inclination" to
engage in the illegal activity is not adequate proof of predisposition. (2) Jury hostility maybe be hostile to defense and not evaluate properly.
D. INTOXICATION - alcohol, drugs, any substance
1. Voluntary Intoxication
Defined as voluntary/without duress; need not be intentional
a. Defense to Specific Intent Crimes
Voluntary intoxication evidence may be offered when D is charged with a crime that
requires purpose (intent) or knowledge to establish that the intoxication
prevented the D from formulating the requisite intent. Thus, it may be a good
defense to specific intent crimes, but usually will not be a sufficient defense to

general intent crimes. The defense is not available if the D purposely becomes
intoxicated in order to establish the defense.
b. No Defense to Crimes Requiring Malice or Recklessness
Not a defense to crimes requiring only malice, e.g., common law murder, recklessness,
or negligence, or crimes of strict liability.
Example After drinking heavily, A breaks into a house wrongly thinking it is her
own. When surprised by B, the owner, A reacts with force, beating B with her
fists. While driving home A is cited for speeding. Will A have a defense of
intoxication: to burglary? (Yes, if as a result she did not know that the house
belonged to B or did not have the intent to commit a felony therein); to battery?
(No, because as defined battery may be the result of recklessness); to speeding?
(No, because speeding is a strict liability offense).
2. Involuntary Intoxication
Intoxication is involuntary only if it results from the taking of an intoxicating substance
(i) without knowledge of its nature, (ii) under direct duress imposed by another, or
(iii) pursuant to medical advice while unaware of the substance's intoxicating effect.
May negate a mens rea element.
Involuntary intoxication may be treated as mental illness, in which case a D is entitled to
acquittal if, because of the intoxication, she meets whatever test the jurisdiction has
adopted for insanity. (Basis for insanity)
3. Relationship to Insanity
Intoxication and insanity are two separate defenses. However, continuous, excessive
drinking or drug use may bring on actual insanity. Thus, a D may be able to claim both an
intoxication defense and an insanity defense.
E. INFANCY
1. Common Law - Rule of sevens: 1) Under Seven - No Criminal Liability; 2) 7 - 14 Rebuttable Presumption of No Criminal Liability; 3) Over 14 - Adult.
2. A number of modern statutes have abolished the presumptions of the common law and
have provided that no child can be convicted of a crime until a stated age is reached,
usually 13 or 14.
3. Juvenile Delinquency
Juvenile court generally has exclusive jurisdiction over children under a certain age, and
concurrent jurisdiction (with the criminal courts) over older children. In most jurisdiction,
the common law immunity rules for infants do not apply in these courts because the
primary goal is rehabilitative rather than punishment.
F. INSANITY - legal term, total defense
Procedure: tough sell. Client must appear competent. Most jurisdictions require notice of defense
-> court order evaluation. Bifurcated hearing.

1. Formulations of Insanity Defense


a. M'Naghten Rule/Test
Right/wrong test. If you knew it was wrong, you fail regardless of your ability to control
your actions. COGNITIVE TEST
1) Elements
The traditional M'Naghten Rule provides that a D is entitled to acquittal if the proof
establishes that:
a) A disease of the mind
b) Cause a defect of reason
c) Such that the D lacked the ability at the time of his actions to either:
(1) Know the wrongfulness of his actions; or
(2) Understand the nature and quality of his actions.
2) Applications
a) Defendant with Delusions
Example: A, because of a mental illness, believed B wanted to kill him. A killed B. Is
A entitled to acquittal on insanity grounds under the M'Naghten Rule? No. Even
if A's delusion had been accurate, he would not have been legally entitled to kill B
simply because B wanted to kill him.
b) Belief that Acts are Morally Right - no right to acquittal
c) Inability to Control Oneself - No Defense
b. TESTS FOR INSANITY
1. Irresistible Impulse Test - D entitled to acquittal if he was unable to control his actions or
to conform his conduct to the law. Need not come upon D suddenly.
2. Durham (or New Hampshire) Test - D entitled to acquittal if he proves his crime was the
product of mental disease or defect. A crime is a product of the disease if it would not have
been committed but for the disease. Broader than either the M'Naghten or irresistible impulse
test.
Understand how Durham test differs from the other two - why psychiatrists given too much
power, why used to often?
3. ALI - Model Penal Code Test (PREVAILING)
Under this test, the D is entitled to acquittal if the proof shows that he suffered from a mental
disease or defect and as a result lacked substantial capacity to either: (i) appreciate the
criminality of his conduct; or (ii) conform his conduct to the requirements of law. This test
combines the M'Naghten and the irresistible impulse tests by allowing for the impairment of
both cognitive and volitional capacity. Highly praised, the A.L.I. test is rapidly becoming the
most popular formulation.

c. EXCLUSION OF PSYCHOPATH
Some jurisdictions exclude psychopaths from def'n of mental illness by saying that repeated
antisocial behavior is not insanity. There is a difference of opinion as to whether this
behavior caused by a disease.
d. DIMINISHED CAPACITY - did not have the particular mental state (mens rea) required for
the crime charged. Most states recognizing this defense limit it to specific intent
crimes. PARTIAL DEFENSE.
2. DIFF. BWTN INSANITY AND COMPETENCE
Insanity is an excuse. Competence refers to one's ability to go through trial. Competence test: (1)
Can the person understand the proceedings? (2) Can he participate/assist in the proceedings?
Hearing for competency -> mental hospital 99.9%. Usually temporary, until competency
restored. Decided by judge -> there will be no trial.
Insanity - not at time of trial but at the time of crime. Moral measure as opposed to capacity
measure. Jury or judge decide. If held, then not guilty of any crime. Only reach insanity stage
after person ahs been declared competent. (virtually never used). When used, usually not
disputed. Little evidence for successfully faking insanity. People tend to stay longer in mental
hospital than if convicted of crime. Recidivism rate about the same as thouse found guilty.
3. Guilty But Mentally Ill (GBMI)
New on the horizon. Michigan. Serves part of sentence in a mental institution, and finish time in
prison. Like a special verdict. Directs D to a mental hospital. Intended as an alternative to finding
people not guilty by reason of insanity. Has not had intended effect.
Has not changed # of people found guilty or not guilty, just increased percentage of guilty sent to
mental hospital first.
4. Effect of Not Guilty by Reason of Insanity Verdict
60 - 90-day obsevation period. Must be released or kept under civil commitment laws.
SUPPLEMENT
1. Overworked participants and overcrowded facilities. - the system adapted to resource limits by
relying on many shortcuts through the criminal justice process.
2. While discretion is subject to abuse, it provides the actors with a flexible process to deal with
both unusual facts and inadequate resources. Some jurisdictions attempting to limit discretion by
given police officers written instructions with how to deal with certain situations.
3. Plea bargaining accounts for 90% of cases.
4. Misdemeanor - usually punishable by jail sentence less than a year.
Sixth Amendment - right to counsel if subject to incarceration.

Steps: complainant - citizen or police; complaint; magistrate; probable cause; arrest warrant;
summons/citation or arrest; station house bail; professional bail bondsperson; preliminary
examination - notified of charges and rights, bail addressed, date for prelim hearing; prelim
hearing - adversarial, if probable cause, then bound over to grand jury; grandjury true bill or
indictment or prosecutor information - if jurisdiction does not utilize grand jury or defendant
waives indictment; test - crime committed and defendant committed it; presentment - formal
allegation that a named person or business has committed a crime; subpoena, contempt of court,
immunity - person cannot use 5th Amendment grounds; arraignment - chance to plead; pretrial
conference - by motion of party or the court, defendant's counsel must be present though
defendant need not be; Trial - motions, discovery, jury selection, swearing in, initial jury
instructions, opening statement, prosecution's case, defense motion to dismiss, defense's case,
rebuttal and surrebuttal evidence, closing arguments, jury instructions, jury deliberations,
announcement of jury verdict; sentencing hearing; direct appeal in most states also rehearing
before same appellate court (perhaps en banc); collateral attack; and executive clemency.
presentment = indictment
grand jury indicts in 95% of cases brought to it
? offended by use of the word "pardon" - to exonerate the innocent
? when and what acceptable for collateral attack. ? Supreme Court declared limit on time for
raising review of even exculpatory evidence?
CRIMES AND TORTS - Lafave and Scott
Criminal law is a subset of tort law. Aim of criminal law - punish harmful results of conduct.
Tort law - compensate someone injured for the harm he has suffered. Consent of injured - may
be a defense in torts but often not in criminal law. Similarities: proximate cause, intervening
cause, duty to act, self-defense.
"preponderance of evidence" vs. "beyond a reasonable doubt"
criminal law is slower to borrow from the civil statutes than the civil law is to borrow from
criminal statutes.
nulla poena sine lege - public policy alone is not sufficient to make conduct criminal, in the absence
of a direct legislative advance warning.
B. DEFINING CRIMINAL CONDUCT
4. SOME LIMITS ON THE CREATION OF CRIMINAL CONDUCT:
SUBSTANTIVE DUE PROCESS
LaFave & Scott, pp. 148-155. Criminal laws may be invalidated because the conducts bears no
substantial relationship to injury to the public.
A statute may also be struck down because it contains no mens rea requirement: it punishes
conduct innocently engaged in without any sort of bad state of mind, or it may be that the statue
punishes mere status or condition instead of requiring some specific act of omission to act.

Deline in the court-asserted control over legislative policy begain in 1934 with Nebbia v. New
York, upholding the conviction of a grocer under a statute prohibiting the sale of milk under
established prices. In Nebbia, the court held "that the legislature is primarily the judge of the
necessity of such an enactment." The Supreme Court is now must reluctant to strike down
statutes on no relation to public injury.
If Court strikes down on "void for vagueness" or "equal protection grounds," a state legislature
can revisit the proscribed conduct. If finding that it violates "pure" substantive due process, then
insulates prohibitive conduct from future legislative restriction. However, state courts have still
struck down statutes as no substantial relation to injury to the public.
p. 152 State courts sometimes strike down a statute on the basis of a finding that the evil as
perceived by the legislature does not exist (sell imitation cream). 2. those person at whom the
statute is directed will escape detection, while others, engaged in innocent pursuits, are likely to
be deterred. (sell cattle hides without brands - not capture the theifs). 4. Pass on alternative
means to protect the public interest - clear labeling is better than prohibiting sale of imitation
cream. 5. true purpose is to serve a special interest group rather than the public 6. Police power
may not be utilized to absolutely prohibit a "legitimate" business.
p.154 Statutes Covering Harmless Conduct. For example, an offense for any jeuvenile to be on
the street past curfew, or possession by anyone of hypodermic needles and syringes without a
doctor's authorization, or sell magazines without their covers (to collect fraudulent refund on
covers). All of these held unconstitutional. Too sweeping in encompassing activity which is
wholly innocent.
Class Notes 9-8-94
Vagueness - constitutional limit on how criminal status are written. Vagueness is a way of
attacking actual statute itself. Standard used in aplying vagueness statute - judicial gloss on
statute. Common understanding test - would reasonable person understand statute.
Jonas v. Blandhi - p.B14 of Supplement - professor accused of cursing over the phone - Did not
make the phone call, therefore not guilty. Initiator of the call has intent.
This statute is also vague - no definition of the terms in the statute. It is also overbroad on First
Amendment grounds.
Principle of Construction - each word is meant to mean something different. They should be
mutually exclusive.
In order to amke this activity illegal, you may wish to include intent to offend. This implies who
initiated the phone call.
Bill of Attainder & Ex Post Facto Laws - Article I Sect. 9 and 10.
Ex post facto - retrospective laws. Not permitted. Apllied to conduct permitted before the law.
TWO ELEMENTS: 1) retrospective application 2) disadvantageous to criminal accused.
Legislature cannot pass law today to make activities of yesterday a crime. Citizen responsible for
law at time of his actions. Punishment is rendered as it was at the time of illegality. "Photograph
of laws at time of crime."

Also cannot change retroactively procedures - only those procedures that protect a substantial
right - to disadvantage of the criminal accused. If legislature passes a new jury law, that law
would not apply to illegalities performed prior to the passage of the law.
Parole laws do not violate ex post fact provision.
If act is not harmful to the criminal, then ex post facto does not apply.
?Does ex post facto apply in judicial decisions?
Bills of Attainder - legislative punishment of people who are readily identifiable. Cannot single
out group or individual for punishment. Bill of Attainder still is not identifiable. If purpose of
the bill is punishment, it is a bill of attainder. Not applicable to procedural matters. (Originally,
death sentence passed by legislature without benefit of trial - legislative decree - Struck Down)
Nixon - Congress passed law that presidents' papers must be taken. Nixon claimed Bill of
Attainder - Nix claimed impetus of the bill was punishment. Supreme Court did not find for
Nixon on issue of who owns presidential papers. Stated if Nixon had a financial loss,
compensation would be made.
III. THEORIES OF PUNISHMENT.
SIX MOST COMMON RATIONALES:
1) retribution - societal revenge and expiation (atone for misdeeds through punishment). Still
commands the most public support. Claimed to suppress acts of private vengeance. May serve as
check against tyranny - you should not be punished unless you did something wrong.
2) restraint - also called incapacitation. Isolate the offender or disable him from offending.
3) rehabilitation - correction or reformation.
4) special deterrence - exposing a criminal to sufficiently distasteful punishment, that individual
will lose the desire to commit crimes.
5) general deterrence - punishment will hopefully deter other would-be criminals from
committing crimes.
6)public education - inform the general public of what conduct is deemed socially unacceptable.
Rehabilitation - most in conflict with the other rationales.
Deterrence - important in cases like income tax evasion where system relies on self-assessment.
p. 29 TREND - jurisdictions adopt sentencing schemes which place the greatest emphasis upon
the nature of the crime which was committed and comparatively little upon the characteristics of
the particular offender - mitigates good behavior argument for parole.
REGINA v. DUDLEY AND STEPHENS, Queen's Bench Division, 14 Q.B.D. 273 (1884).
(P.A66-70 of supplement). Appeal of jury's special verdict finding Dudley (D) and Stevens (D)

guilty of murder of Parker, with whom they were stranded on the high seas in a lifeboat, in order
to survive off Parker's remains after having run out of food and water.
FACTS: Dudley (D), Stephens (D), Brooks, and Parker, crew members of an English yacht, were
cast adrift on the high seas 1,600 miles from land in an open lifeboat. They had no water and
two one-pound tins of turnips. After 12 days adrift, they were without food. Dudley (D) and
Stephens (D) suggested to Brooks that one of the four may have to be sacrificed so that the
others might survive. Brooks dissented, and Parker, a 17-year old boy, was never consulted. On
the 20th day, Dudley (D) and Stephens (D) killed Parker, who was too weak either to resist or
assent. Four days after Parker's death, the surviving three were rescued. They would not have
survived had they not fed off Parker's remains.
ISSUE: Was the homicide excusable by the necessity of saving some of the crewmen?
HOLDING AND DECISION: No. An innocent person may not be killed in order to save the
life of another. Where the victim has not assaulted or otherwise endangered the killer, the killer
has not, by necessity, been placed in a position which permits him to kill the innocent victim.
The extreme necessity of hunger does not justify larceny, nor can it justify murder. While,
generally, the preservation of one's own life is a duty; in some cases, the highest duty may be to
sacrifice it. Neither can the temptation caused by hunger be called an excuse.
COMMENT: While this case actually discusses a defense to murder, necessity (which here did
not excuse the murder), the case appears here in the casebook more for its moral discussion of
why the defendants, unwillingly placed in a tragic situation, must be punished for their act. The
court notes that "Law and morality are not the same, and many things may be immoral which are
not necessarily illegal," but that law would be divorced from morality if the temptation to kill,
which arose, could be an excuse for the actual killing. Even if the temptation were a valid excuse,
who is to determine who must die so that the others might live? (The death sentence was later
commuted by the crown to six months' imprisonment.)
COHEN AND HALL SUPPLEMENT, PP. A18-29, 40-65
p.A18 In most criminal cases that proceed to trial, the defendant will be found guilty and will
serve some kind of sentence.
p.A19 In addition to sentencing guidelines, two current proposals are "Truth in Sentencing" (the
sentence a criminal serves should be about the same as that announced by the judge) and "Three
Strikes You're Out," (third time violent offenders should receive a sentence of life without
possibility of parole.
Legislature may establish a mandatory minimum sentence or even set a mandatory sentence that
must be imposed. RECENT SENTENCING SYSTEMS HAVE CONFINED THE JUDGES'
DISCRETION BY ADOPTING VARIOUS GUIDELINES.
In most situations, the plea bargainer is deprived of the right to appeal the sentence. See, e.g., 18
U.S.C 3742(c).
p.20 When a jurisdiction's jails and prisons are full, sentencing decisions must reflect this reality.
Sentences can be determinate and indeterminate. A sentence of 5-10 years is indeterminate.

p.22 Rehabilitation assumes the accused is "sick" and can be "cured" by the proper dose of
punishment.
p. 24 Incapitation, rehabilitation, and deterrence - require you to predict future behavior and
dangerousness.
p.27 "Equal Protection" has infrequently been successful in challenging sentences or sentencing
procedures. The key is finding an impermissible discrimination.
The Eighth Amendment's cruel and unusual punishment clause is used to challenge the
severity of a sentence. The offender claims that a particular sentence is so harsh that it is
unconstitutional. These claims are rarely sucessful. It is not clear to what extent the eight
amendment embraces a requirement that sentences be proportionate to the offense.
A law violates the ex post facto provision if it is (1) retrospective and (2) disadvantageous to the
offender.
p. 28 Any law that causes a defendant to suffer a disadvantage, however, does not offend this
provision. The Supreme Court has distinguished between disadvantageous changes affecting
procedure and those altering substantial personal rights. The ex post facto provision was not
designed to limit the legislature's prerogative of altering both remedies and procedures which do
no involve matters of substance. Increasing the penalty for a crime and imposing it retroactively
violates the ex post facto guarantee. Altering the method used to determine whether the death
penalty was imposed was constitution. It did not change the "quantum of punishment" attached
to the crime. New Florida procedures did not change the definition of the crime or the amount
of proof necessary for guilt.
Parole - must satisfy a list of requirements like hold a job and maintain regular contact with
payroll officer.
Halfway house, split confinement
p. 42 PROBATION - suspend execution and suspend imposition. In the case of a probation
violation, the court may be bound by the original prison sentence it imposed in the suspended
execution model, but has no such limits in the suspended imposition model. Probationers enjoy
a conditional liberty that cannot be taken away without certain due process procedures.
p.43 FACTORS a FEDERAL COURT must consider in determining whether to impose a
FINE: (1) D's ability to pay, (2) burden that payment of the fine will impose on both the D and
his/her dependents, (3) whether restitution will be made by D to victim.
SCANDANAVIAN "DAY FINE" - fine a person the equivalent to a certain # of days pay for
work, e.g., 100 days. This system is fair to both rich and poor, since it punishes them
proportionately and equally in terms of relative loss.
FORFEITURE OF PROPERTY - another sentencing option.
RICO - bars a peson from receiving income from racketeering or illegal debt collection activities.
18 U.S.C. 1962. SC has upheld the forfeiture of drug proceeds used to pay for defense counsel.
Book proceeds restrictions against criminals violate First Amendment.

RESTITUTION - another alternative to sentencing.


DIVERSION - if does not violate law again, or meets community service requirement, charges
will not be filed or will be dismissed.
PROCEDURES - often the rules of evidence do not apply at sentencing hearings, making
hearsay evidence both admissible and frequently used.
p.49 Minnesota guidelines: 1) presecriptive - establish its own policies; 2) de-emphasize
imprisonment for property crimes, and emphasize for violent crimes; 3) in order to attack
sentencing disparities, established narrow sentencing ranges (e.g., 30 - 34 months, 50 - 58
months); 4) adopted "just deserts" as the governing premise of its policies concerning who
received prison sentences; 5) had to take existing prison capacity as given, thus if lengthened one
sentence area would have to shorten in another. 6) forbade consideration of education,
employment, marital status, etc. in sentencing decision - to avoid discrimination against
minorities, women, or low income groups.
FEDERAL SENTENCING GUIDELINES - requires the calculation of THE OFFENSE
LEVEL and the CRIMINAL HISTORY CATEGORY.
p. A54 "real offense" sentencing - permits Courts to consider "relevant conduct" of criminal and
reasonably foreseeable acts of others involved in the crime, whether or not these other acts were
charged as crimes.
LAFAVE & SCOTT, pp.177-184
The Eighth Amendment prohibits the federal government from imposing cruel AND unusual
punishments for federal crimes; the Due Process clause of the Fourteenth Amendment prohibits
states from inflicting such punishment for state crimes.
The prohibition has three aspects: (1) limits the methods which may be used to inflict
punishment; (2) it limits the amount of punishment which may be prescribed for various
offenses; and (3) it bars any and all penal sanctions in certain situations - invalidate California law
making it a crime to be "addicted to narcotics." Robinson v. California. Public drunkenness
distinguished from status crime in Powell v. Texas.
The scope of the Eighth Amendment "must draw its meaning from the evolving standards of
decency that mark the progress of a maturing society."
Castration is illegal, ? voluntary castration in exchange for no sentence legal?
Gregg v. Georgia 1976 the U.S. Supreme Court reinstated the death penalty. The court
emphasized three factors: 1) long history of use for murder in England and the U.S.; 2) large
proportion of the population regard it as appropriate and necessary; 3) serves purposes of
retribution and deterrence.
p.178 It is unconstitutional for a state to mandate the death penalty for a certain degree or
category of murder; or for those murders unaccompanied by a few specified mitigating
circumstances; or for a sentencing judge to disregard as a matter of law relevant mitigating
circumstances.

capital punishment must "be imposed fairly, and with reasonable consistency, or not at all," for
"a consistency produced by ignoring individual differences is a false consistency."
Courts have seldom held a punishment may be cruel due to excessiveness. Courts are seldom to
strike down sentence on proportionality grounds.
p.180 Coker v. Georgia, 1977, The Supreme Court held death penalty unconstitutional for rape.
"It is the first modern decision in which the Supreme Court has relied on disproportionality to
invalidate a punishment under the cruel and unusual punishment clause."
Robinson v. California, 370 U.S. 660, 82 S.Ct. 1417 (1962). Appeal from conviction for narcotic
addiction. FACTS: A california statute makes it a crime for a person to be addicted to the use of
narcotics. The evidence against Robinson (D) was a police officer's testimony that Robinson (D)
had scars, discolorations, and needle marks indicating his frequent use of narcotics.
ISSUE: Is a law which makes narcotics addiction a criminal offense constitutional?
DECISION: No. Narcotic addiction is an illness which may be contracted innocently or
involuntarily. Like mental illness, leprosy, or venereal disease, a state could establish a program of
compulsory treatment, involving quarantine or confinement for narcotics addicts. However, a
law which makes a criminal offense of narcotic addiction inflicts a cruel and unusual punishment
in violation of the 8th and 14th Amendments. The California (P) statute imprisoning a person
addicted to narcotics, even though he has never used any drug within the state, or been guilty of
any irregular behavior there, is unconstitutional.
CONCURRENCE: Confinement for the purposes of punishing, rather than curing, drug
addiction cannot be upheld.
DISSENT: The court has remove the state's power to deal with the recurring case where there is
ample evidence of use but no evidence of the precise location of use.
COMMENT: punishment hear is not cruel, but the conduct should not be the subject of
criminal sanction.
POWELL v. TEXAS, 88 S.Ct. 2145, (1968). Appeal from conviction for public drunkenness.
FACTS: Powell (D), a chronic alcoholic, was found guilty of being drunk in public. He was fined
$20. D argued that his appearance in public was not of his own volition, and that to punish him
for his illness would be cruel and unusual in violation of the 8th Amendment as applied to states
by the 14th Amendment. The medical profession has not yet determined whether alcoholism is
an illness, is physically addicting, or merely psychologically habituating. On appeal, the court
ruled that chronic alcoholism was not a defense to the charge and fined D $50.
ISSUE: Can punishment for public drunkenness be constitutional?
DECISION: Yes. D was not convicted for being a chronic alcoholic, but for being drunk in
public. Here the conviction protects public safety and health. Facilities for treating indigent
alcoholics are woefully lacking. A short time in jail will enable an alcoholic to sober up. Some
actus reus is required before criminal penalties may be inflicted.
Because penal incarceration has stringent time limits, it is probably preferable to an unlimited
civil commitment when no treatment is available.

RULE: Chronic alcoholics do not suffer from such an irresistible compulsion to drink and to get
drunk in public, that they are utterly unable to control their performance of either or both of
these acts, and thus cannot be deterred at all from public intoxication.
GREGG v. GEORGIA, 428 U.S. 153 (1976). Gregg (D) was found guilty of murder and
sentenced to death. Furman 1972 had previously struck down the death penalty for capricious
and arbitrary manner it was imposed. Here the court upheld the constitutionality: it is not "cruel
and unusual" per se.
RATIONALE: 1. Cap. punishment is essential to a society which requires reliance on legal
process instead of self-help.
2. Statistical attempts to evaluate the deterrent effect have been inconclusive, the resolution of
this complex factual issue rests with the state legislatures.
3. Under Georgia's sentencing procedure, the jury's discretion is always circumscribed by
legislative guidelines. The death penalty is available for only six categories of crimes: murder,
kidnapping for ransom or where the victim is harmed, armed robbery, rape, treason, and aircraft
hijacking. After a verdict of guilty, a separate sentencing hearing is conducted, and the jury must
find that one of ten specified aggravating circumstances exists before imposing the death penalty.
This satisfies the concerns of Furman as to capricious and arbitrary administration.
DISSENT (Brennan): Our civilization and the law have progressed to the point that punishment
of death, for whatever crime and under any circumstances, is "cruel and unusual."
MCCLESKEY V. KEMP, 107 S.Ct. 1756 (1987). The death penalty is not unconstitutional
because of statistics demonstrating a tendency towards racial bias in its application. FACTS:
McClesky (D), a black man, was convicted of killing a white police officer. The jury
recommended the death penalty which the court imposed. McCleskey (D) appealed, producing
statistical studies tending to prove that the penalty was imposed much more frequently on blacks
than whites, particularly when the victim was white. The penalty was affirmed. The district court
refused to grant habeas corpus, and the court of appeals affirmed. The Supreme Court granted
certiorari.
DECISION: The death penalty is not unconstitutional because of statistics demonstrating a
tendency toward racial bias in its application.
RATIONALE: The death penalty is not unconstitutional because of statistics demonstrating a
tendency towards racial bias in its application. To prove an Equal Protection violation, a person
must prove he was the victim of purposeful discrimination. Evidence of statistical tendencies
does not prove that the decision makers in his case were biased. In terms of the 8th Amendment
analysis, the discretion that this Court has mandated in the application of the death penalty
necessarily leaves room for bias. However, the only other alternative, mandatory application of
the penalty, is antithetical to the fundamental role of discretion in our penal system. Finally, to
hold that statistical evidence of bias in death penalty application makes it unconstitutional would
open the door to a broad range of challenges to various aspects of the criminal justice system,
which would impose unacceptable costs thereon. Here, as the only challenge McCleskey (D) has
made is based on statistical analysis, his challenge is insufficient. Sentence affirmed.
DISSENT: the proper analysis is not whether an arbitrary sentence has been imposed, but
whether there is an unacceptable risk thereof. The studies here demonstrate this to be the case.

DISSENT: In terms of equal protection analysis, one challenging a particular system need only
show that the totality of relevant facts gives rise to an inference of a discriminatory purpose, a
burden McCleskey (D) has met.
BURDEN OF PROOF -- MULLANEY v. WILBUR, 421 U.S. 684 (1975). FACTS: Petition
for certiorari by the prosecution due to district and appellate courts allowing habeas corpus
relief. Wilbur (D) fatally assaulted Herbert after Herbert's homosexual advance. The defnse
argued that the homicide was not unlawful since D lacked criminal intent or, alternatively, that at
most the homicide was manslaughter rather than murder since it occurred in the heat of passion
provoked by the assault. The state of Maine, where the homicide occurred, requires that the
defendant charged with murder prove he acted "in the heat of passion on sudden provocation" i
order to show lack of malice aforethought and thus reduce the homicide from murder to
manslaughter. D was convicted of murder and appealed to the Maine Supreme Court, which
rejected D's contentions. D then filed a writ of habeas corpus with the federal district court on
the basis that the Maine rule violates due process because the prosecution is not required to
prove beyond a reasonable doubt every fact necessary to constitute the crime charged but is
permitted to rely on implied malice aforethought if D fails to prove he acted in the heat of
passion on sudden provocation. The court of appeals affirmed the district court's ruling in favor
of D and the U.S. Supreme Court granted certiorari.
ISSUE: Does the Maine requirement that D prove he acted "in the heat of passion on sudden
provocation" constitute a denial of due process?
DECISION: Yes. Judgment for D affirmed.
1. Here the state argued that there was no denial of due process, since homicide and the presence
or absence of heat of passion on sudden provocation pertained merely to the punishment.
However, a state could undermine a D's right to due process by eliminating the requirement that
prosecution prove beyond a reasonable doubt the elements of the crime by instead redefining
these elements and characterizing them simply as factors that bear solely on the extent of
punishment.
2. Requiring the defendant to prove the critical fact in dispute increases the likelihood of an
erroneous conviction. The prosecution must prove beyond a reasonable doubt the absence of
the heat of passion on sudden provocation when the issue is properly presented in a homicide
case.
In PATTERSON v. NEW YORK, 432 U.S. 197 (1977), the Supreme Court held that an
affirmative defense to a charge of murder provided by New York law (i.e., that defendant acted
under the influence of an extreme emotional disturbance for which there was a reasonable
explanation or excuse) was a considerably expanded version of the common law defense of heat
of passion on sudden provocation. The burden of proving such an affirmative defense could be
placed on the D without constituting a denial of due process. A dissenting opinion, however,
arged that Patterson really was indistinguishable from Mullaney.
UNITED STATES v. PARK, 421 U.S. 658 (1975).
FACTS: Appeal from reversal of conviction for adulterating food in violation of federal law.
Park (D) was president of Acme Markets, Inc., a food chain with 874 outlets. Rodents
contaminated food stored at an Acme warehouse. Federal inspectors discovered this and wrote
to D. Three months later, although there was some improvement, rodent contamination was still

discovered. Acme and D were charged with violating laws prohibiting adulterating food by
contamination. Acme pleaded guilty to all counts and D pleaded innocent. At trial, evidence
showed that D had delegated responsibility for sanitary conditions to a vice-president in whom
he had confidence. D was convicted at trial. The circuit court of appeals reversed the conviction.
The U.S. appealed.
ISSUE: May a corporate official be prosecuted for a cime solely by virtue of his position of
supervisory responsibility over the corporate area wherein the infraction arose?
HELD: Yes. The circuit court is reversed and conviction upheld.
1. The federal law involved here dispenses with the need of proving conscious wrongdoing.
2. Here. Acme's operations touch phases of the lives and the health of people which, in the
circumstances of modern industrialism, are largely beyond self-protection. In the interest of the
larger good, the federal law puts the burden of acting on a hazard upon the person otherwise
innocent but standing in responsible relation to remedy the public danger.
3. The only way a corporation, such as Acme, can act is through people.
4. The liability under the act is limited to corporate employees who have a "responsible share" in
the furtherance of the transaction which the statute outlaws.
5. Moreover, the principle is recognized that a corporate agent, through whose act, default, or
omission the corporation becomes guilty of a crime, is himself guilty of the crime.
6. Finally, the Act imposes not only a positive duty to seek out and remedy violations but also a
duty to implement measures that will ensure that violations will not occur. However, the Act
does not require that which is objectively impossible. Ultimately, the government must prove
beyond a reasonable doubt the defendant's guilt.
7. The jury charge that D contends is misleading faily advised the jury that in order for D to be
guilty, he had to be in a responsible relation to the situation. The circuit court's reversal of D's
conviction is reversed.
DISSENTS (Stewart, Marshall, Powell): The jury instruction given by the trial judge could have
allowed the jury to find D guilty purely due to the position he occupied in the corporate
structure.
2. Here we are dealing with criminal penalties, not civil forfeiture. The conviction here is alient to
fundamental principles of law.
3. The jury should have been clearly instructed that D had to be engaged in wrongful conduct at
least amounting to common law negligence. There was no such instruction in this case.
COMMENT: The imposition of corporate liability does not affect the criminal responsibility of
the actual perpetrator. Hence, both the corporation and the guilty employee may be convicted of
the offense.

HOMICIDE. The killing of one human being by another human being. At common law,
homicides are justifiable, excusable, or ciminal; however, modern statutes tend to abandon this
categorization effort.
Justifiable homicides are those killings authorized by law. The killer is regarded as simply doing
what the law permits. a) execution of prisoner by court order; b) killing by a police officer or
private person to prevent the victim from committing a felony by violence or surprise; c) killing
by a police officer or private person in suppressing a riot; d) killing to effect the arrest of the
perpetrator for a felony; e) killing by one who was feloniously assaulted to save himself from
death or great bodily harm.
Excusable Homicide: those in which the killer is not necessarily without fault but where the
circumstances do not justify the infliction of the normal punishment for homicide. At common
law, excusable homicides are penalized by forfeiture of goods and chattels. Excusable homicides
are of two types:
(1) Compare - killing in self-defense as justifiable. One engaged in legitimate self-defense
who accidentally kills the aggressor commits excusable homicide, whereas a killing
is justifiable if it is intentional, and done under circumstances in which the law authorizes a
killing in self-defense.
(2) Killings in self-defense upon a sudden affray. This involves killing another in the course
of a sudden mutual fight in order to save oneself from a reasonably apparent danger of death or
great bodily harm.
Criminal homicide. Killings without justification or excuse are criminal homicides, either murder
or manslaughter.
Classification under modern law. Under modern statutes and decisions, excusable homicides
are no longer punished, and so the distinction between excusable and justifiable killings is of no
significance. Rather, homicides are now simply regarded as lawful or unlawful (although there are
a number of different offenses which may be punished as criminal homicides).
ONE METHOD TO ANALYZE HOMICIDE SITUATIONS: (1) Check to see if elements are
present for murder, voluntary manslaught, and involuntary manslaughter. Analytical steps:
1. Is it reasonably likely that the defendant had, at the time of the killing, any of the states of
mind that will suffice to establish malice aforethought? If so, the killing may be murder.
2. If there is an applicable statute, is it reasonably likely that the crim can be elevated (from what
would otherwise be second degree murder) to first degree murder?
3. If there is a reasonable likelihood that the killing might be murder, is there any basis for
finding adequate provocation and thereby mitigating the crime to voluntary manslaughter?
4. If there is no reasonable likelihood that the killing might be murder, do the facts support a
finding of involuntary manslaughter?
5. If there is any possibility that the defendant may have committed one of the above offenses,
are there any problems of causation? Consider all of the following possibilities: a) "factual"
causation problems? b) "proximate" causation problems?

c) special causation problems posed by the "year-and-a-day" rule?


HEAT OF PASSION
Voluntary Manslaughter. A killing that would otherise be murder but that was committed in
response to adequate provocation is committed without malice aforethought and is
therefore voluntary manslaughter. Gnerally, the provocation must be such as might render
ordinary persons of average disposition liable to act rashly, without deliberation, and from
passion rather than judgment.
Elements of adequate provocation: 1. reasonable provocation - not enough D provoked if
reasonable person would not have been; 2. actual provocation; 3. absence of reasonable cooling
period; 4. No actual cooling off.
Model Penal Code position. The Model Penal Code provides that the reasonableness of the
disturbance which reduces a killing to manslaughter is to be determined from the viewpoint of a
person in the D's position under the circumstances as the D believed them to be. This allows the
court to take into account some of the D's special characteristics, but it is not clear where the
line is to be drawn.
Particular situations. 1. Words alone. The traditional and majority view has been that mere
words, no matter how insulting, are not adequate provocation.
2. Battery. Blow must be violent and painful to be sufficient provocation.
3. Assault. it may constitute provocation, expecially in aggravated cases.
4. Illegal arrest. Unlawful arrest can constitute provocation, particularly where the D is aware of
unlawful nature of arrest.
5. Adultery. It is well established that the discovery of one's spouse in the act of committing
adultery is sufficient provocation. Moreover, the modern trend is to extend this rule beyond
situations where one spouse actually catches the other in the act. Thus, some courts find
sufficient provocation where the D is told of his or her spouse's adultery.
6. Mutual quarrel or combat. If two persons voluntarily engage in a fight in the course of
which one is killed, the homicide is only manslaughter. Conceptually, the suddenness of the
affray, rather than any specific action by the victim, is the provocation.
7. If she was mistaken as to the existence of the provocation but her mistaken belief was a
reasonable one, the killing should still be mitigated.
8. Defendant intends to kill provoking party. If the D killed someone else, either by accident
or because she was mistaken as to who provoked her, the killing is still only voluntary
manslaughter.
9. Defendant intends to kill nonprovoking party. the killing is not reduced to manslaughter.
10. If the subject of the provocation is a close relative of the D, rather than the D herself, the
provocation may still be adequate. And while the case law suggests that this rule would not be
applied if the subject of the provocation were a distant relative or a mere friend, modern courts,

reluctant to hold offered provocation inadequate as a matter of law, might well permit such cases
to go to the jury.
MURDER -DELIBERATE OR PREMEDITATED KILLING
In the common law, murder is the unlawful killing of another human with malice
aforethought.
"Malice aforethought" is sometimes defined as the intention to kill, actual or implied, under
circumstances who do not constitute excuse or justification or mitigate the offense to
manslaughter. The intent to kill is "actual" where the D consciously desired to cause death, and
"implied" where the defendant intended to cause great bodily harm or where the natural
tendency of his behavior was to cause death or greath bodily harm. No ill will or hatred of the
victim need be shown.
Malice aforethought encompasses several different mental states; absent evidence of
provocation, malice aforethought exists if the D entertained any one of the following states of
mind:
1) Intent to kill. In most cases, the intent to kill must be inferred from the surrounding
circumstances, as only rarely will the accused have announced his mental state. Often, however,
the inference can be drawn from the D's conduct. Thus, one who intentionally uses a deadly
weapon (a weapon calculated to produce death or great bodily injury) on another human being,
and thereby kills him, is presumed to have intended the killing. Under the better view, this is not
a mandatory presumption.
2) Intent to inflict great bodily injury. A murder conviction is likewise supported by evidence
that the D intended to inflict serious bodily injury, even though he did not consciously desire to
cause the victim's death.
3) Awareness of a high risk of death or serious bodily injury. Under certain exceptional
circumstances, a D may be guilty of murder where he acted in the face of an unusually high risk
that his conduct would cause death or serious bodily injury. Traditionally, it is said that the risk
must have been such that ignoring it demonstrates an "abandoned and malignant heart."
4) Intent to commit a felony. As will be seen, if the D was in the process of committing a
felony - and therefore had the intent to commit a felony - he has acted with malice aforethought.
This is the basis of the felony-murder rule.
5) Actual awareness of risk. There is some dispute as to whether the D must have been
actually aware of the grave risk involved or whether it is enough that his conduct created that
risk.
6) Proof of malice aforethought. The various states of mind comprising malice aforethought
may be proved by circumstantial evidence.
FIRST DEGREE MURDER. No common law equivalent. Creature of statute. Includes: (1)
Premeditated killings - most states no appreciable time needed. (2) Killing during enumerated
felonies: arson, rape, robbery, burglary, kidnapping, mayhem, and sexual molestation of a child.

SECOND DEGREE MURDER: all killings committed with malice aforethought which are
not specifically made first degree murder.
RECKLESS AND NEGLIGENT KILLING
An unintended killing is involuntary manslaughter if it is caused during the commission of an
unlawful act not amounting to a felony or if it is the result of criminal negligence.
"Misdemeanor Manslaughter" - also supports a felony that, for any reason, will not support
felony murder will be enough. Moreover, some acts, although not criminal under the law, have
been held "unlawful" for manslaughter purposes.

You might also like